Monotone classes (1). A family (mathscr{M} subset mathscr{P}(X)) which contains (X) and is stable under countable unions

Question:

Monotone classes (1). A family \(\mathscr{M} \subset \mathscr{P}(X)\) which contains \(X\) and is stable under countable unions of increasing sets and countable intersections of decreasing sets.

(An)EN CM, AC... CA C An+11A=U An A  M (Bn)neN CM, B B B+1+B= B BEM. = #EN "EN (MC) (MC)

is called a monotone class. Assume that \(\mathscr{M}\) is a monotone class and \(\mathscr{F} \subset \mathscr{P}(X)\) any family of sets.

(i). Mimic the proof of Theorem 3.4 to show that there is a minimal monotone class \(\mathfrak{m}(\mathscr{F})\) such that \(\mathscr{F} \subset \mathfrak{m}(\mathscr{F})\).

(ii). If \(\mathscr{F}\) is stable w.r.t. complements, i.e. \(F \in \mathscr{F} \Longrightarrow F^{c} \in \mathscr{F}\), then so is \(\mathfrak{m}(\mathscr{F})\).

(iii). If \(\mathscr{F}\) is \(\cap\)-stable, i.e. \(F, G \in \mathscr{F} \Longrightarrow F \cap G \in \mathscr{F}\), then so is \(\mathfrak{m}(\mathscr{F})\).

[ show that the families]

E:= {Mem(F): ' := {Mem(F): Mn Fem (F) VF  F} MNE m(F)VNEM(F)}

are again monotone classes satisfying \(\mathscr{F} \subset \Sigma, \Sigma^{\prime}\).]

(iv). Use (i)-(iii) to prove the following.

Monotone class theorem. Let \(\mathscr{F} \subset \mathscr{P}(X)\) be a family of sets which is stable under the formation of complements and intersections. If \(\mathscr{M} \supset \mathscr{F}\) is a monotone class, then \(\mathscr{M} \supset \sigma(\mathscr{F})\).

Data from theorem 3.4

Theorem 3.4 (and Definition) (i) The intersection ni Ai of arbitrarily many o-algebras A; in X is again a

Fantastic news! We've Found the answer you've been seeking!

Step by Step Answer:

Related Book For  answer-question
Question Posted: